drawing a sketch, giving an example, or providing a written description, please indicate themeaning of each of the following shapes.

Drawing A Sketch, Giving An Example, Or Providing A Written Description, Please Indicate Themeaning Of

Answers

Answer 1

For the given shapes, we will draw a sketch

a) A cone

the sketch of the cone will be as follows:

The cone has a circular base of radius = r, and a height of (h) and has a flat surface and curved surface as shown.

b) The diameter of the circle:

The diameter is a line segment (d) that connects two points lying on the circle through the center of the circle

c) The radius of the circle:

The radius of the circle (r) is a line segment that connects the center of the circle and any point lying on the circle

Drawing A Sketch, Giving An Example, Or Providing A Written Description, Please Indicate Themeaning Of
Drawing A Sketch, Giving An Example, Or Providing A Written Description, Please Indicate Themeaning Of
Drawing A Sketch, Giving An Example, Or Providing A Written Description, Please Indicate Themeaning Of

Related Questions

Can you please give me a step by step explanation/solution. Thanks

Answers

The Perimeter of a Rectangle

Given a rectangle of width w and length l, the perimeter is calculated as the sum of the side lengths, that is:

P = w + w + l + l

Or, equivalently

P = 2w + 2l

Hermann was calculating the perimeter of a rectangle and built the expression:

P = x + x + 4x + 4x feet

Note this expression is similar to the first one. This means that the width of the rectangle is x and the length is 4x.

a) We'll draw a rectangle with such dimensions:

b) Assuming the base is the length and the height is the width, the relationship between the base and the height is

4x / x = 4

This means the base is four times the height

c) We are given the perimeter as P = 60 feet.

We need to solve the equation

x + x + 4x + 4x = 60

Simplifying:

10x = 60

Dividing by 10:

x = 60/10

x = 6 feet

The base is 4x = 24 feet.

The base of Herman's rectangle is 24 feet

The height of Herman's rectangle is 6 feet

Evaluate. Assume u > O when In u appears. (In x)96 1 dex X O 96(In x)95+C (In x97 97x +C O (In x)97 +C O (In x)97 97 +

Answers

EXPLANATION

[tex]\int \frac{(\ln x)^{96}}{x}dx[/tex]

Applying subtitution: u=ln(x)

By integral substitution definition

[tex]\int f(g(x))\cdot g^{^{\prime}}(x)dx=\text{ }\int f(u)du,\text{ u=g(x)}[/tex]

Substitute: u=ln(x)

[tex]\frac{du}{dx}=\frac{1}{x}[/tex][tex]\frac{d}{dx}=(\ln (x))[/tex]

Apply the common derivative:

[tex]\frac{d}{dx}(\ln (x))=\frac{1}{x}[/tex][tex]\Rightarrow du=\frac{1}{x}dx[/tex][tex]\Rightarrow dx=xdu[/tex][tex]=\int \frac{u^{96}}{x}\text{xdu}[/tex]

Simplify:

[tex]\frac{u^{96}}{x}x[/tex]

Multiply fractions:

[tex]a\cdot\frac{b}{c}=\frac{a\cdot b}{c}[/tex][tex]=\frac{u^{96}x}{x}[/tex]

Cancel the common factor: x

[tex]=u^{96}[/tex][tex]=\int u^{96}du[/tex]

Apply the Power Rule:

[tex]\int x^adx=\frac{x^{(a+1)}}{a+1},\text{ a }\ne\text{ -1}[/tex][tex]=\frac{u^{96+1}}{96+1}[/tex]

Substitute back u=ln(x)

[tex]=\frac{\ln ^{96+1}(x)}{96+1}[/tex]

Simplify:

[tex]\frac{\ln ^{96+1}(x)}{96+1}[/tex]

Add the numbers: 96+1=97

[tex]=\frac{\ln ^{97}(x)}{97}[/tex][tex]=\frac{1}{97}\ln ^{97}(x)[/tex]

Add a constant to the solution:

[tex]=\frac{1}{97}\ln ^{97}(x)\text{ + C}[/tex]

The answer is D:

[tex]\frac{(\ln x)^{97}}{97}+C[/tex]

A rental car company charges 23.95 per day to rent a car and $0.08 for every mile driven. Nathan wants to rent a car, knowing that:He plans to drive 400 miles.He has at most $440 to spend.Which inequality can be used to determine x, the maximum number of days Nathan can afford to rent for while staying within his budget?

Answers

Answer

Explanation

The total charge on the car = (Charge based on number of days) + (Charge based on the number of miles)

Charge based on number of days = (23.95) (x) = (23.95x) dollars

Charge based on the number of miles = (0.08) (400) = 32 dollars

(The total charge on the car) ≤ 440

23.95x + 32 ≤ 440

23.95x + 32 - 32 ≤ 440 - 32

23.95x ≤ 408

Divide both sides by 23.95

(23.95x/23.95) ≤ (408/23.95)

x ≤ 17.04

Hope this Helps!!!

I need help. I have no idea how to respond this question

Answers

An angle with its vertex on a circle and chord-shaped sides is said to be inscribed. The arc that is inside the inscribed angle and whose endpoints are on the angle is known as the intercepted arc.

A triangle's internal angles are always divisible by 180 degrees.

The central angle of one radian (s = r) subtends an arc length of one radius. One radian has the same value for all circles because they are all alike. The central angle of a circle is measured by its arc, which is 360 degrees, and its radian measure, which is 2π.

Circles:

A circle is a particular type of ellipse in mathematics or geometry where the eccentricity is zero and the two foci are congruent. A circle is also known as the location of points that are evenly spaced apart from the center. The radius of a circle is measured from the center to the edge. The line that splits a circle into two identical halves is its diameter, which is also twice as wide as its radius.

To learn more about circle visit: https://brainly.com/question/17357009

#SPJ9

Sarah is saving money to go on a trip. She needs at least $1975 in order to go. Sarah is mowing lawns and walking dogs to raise money. She charges $25 each time he mows a lawn and $15 each time she walks a dog. I have to Define the variables for the problem and Write an inequality to model this problem

Answers

We're told from the question that Sarah needs atleast $1975, that means that she can either have exactly $1975 or more but not less;

Let x represent the number times she mows a lawn;

Let y represent the number of times she walks a dog;

The inequality can be modelled thus;

[tex]25x+15y\ge1975[/tex]

The function y=f(x) is graphed below. Plot a line segment connecting the points on ff where x=-8 and x=-5. Use the line segment to determine the average rate of change of the function f(x) on the interval −8≤x≤−5.

Answers

Answer:[tex]\begin{gathered} \triangle y=-10 \\ \triangle x=3 \\ Average\text{ rate of change}=-\frac{10}{3} \end{gathered}[/tex]

Explanations:

The formula for calculating the rate of change of a function is expressed as:

[tex]f^{\prime}(x)=\frac{f(b)-f(a)}{b-a}[/tex]

Using the connecting points x = -8 and x = -5 on the graph, this means:

a = -8 = x1

b = -5 = x2

f(b) is f(-5) which is the corresponding y-values at x = -8

f(a) is f(-8) which is the corresponding x-values at x = -5

From the graph;

f(b) = f(-5) = -20 = y2

f(a) = f(-8) = -10 = y1

Determine the change in y and change in x

[tex]\begin{gathered} \triangle y=y_2-y_1=-20-(-10) \\ \triangle y=-20+10=-10 \\ \triangle x=x_2-x_1=-5-(-8) \\ \triangle x=-5+8=3 \end{gathered}[/tex]

Find the average rate

[tex]\begin{gathered} Average\text{ rate of change}=\frac{f(b)-f(a)}{b-a}=\frac{\triangle y}{\triangle x} \\ Average\text{ rate of change}=-\frac{10}{3} \end{gathered}[/tex]

For the grah , draw a line connecting the coordinate point (-5, -20) and (-8, -10)

Solve the system of linear equations using the substitution method. 4x+4y=12x=-2y+8

Answers

Hello there. To solve this question, we'll need to isolate a variable, substitute its expression into the other equation and find both values.

4x + 4y = 12

x = -2y + 8

Plug x = -2y + 8 in the first equation. Before doing so, divide both sides of the first equation by a factor of 4

x + y = 3

-2y + 8 + y = 3

Subtract 8 on both sides of the equation and add the values

-2y + y = 3 - 8

-y = -5

Multiply both sides of the equation by a factor of (-1)

y = 5

Plug this value into the expression for x

x = -2 * 5 + 8

Multiply the values

x = -10 + 8

Add the values

x = -2

These are the values we're looking for.

The solution for this system of equation is given by:

S = {(x, y) in R² | (x, y) = (-2, 5)}

Answer:

         x = -2

         y = 5

Step-by-step explanation:

Solving system of linear equations by substitution method:

      4x + 4y = 12

Divide the entire equation by 4,

        x + y = 3 -------------(I)

             x = -2y + 8 ------------(II)

Substitute x = -2y + 8 in equation (I)

      -2y + 8 + y = 3

      -2y + y + 8 = 3

Combine like terms,

           -y + 8    = 3

Subtract 8 from both sides,

                 -y   = 3 - 8

                -y    = - 5

Multiply the entire equation by (-1)

                 [tex]\sf \boxed{\bf y = 5}[/tex]

Substitute y= 5 in equation (II),

        x = -2*5 + 8

           = - 10 + 8

       [tex]\sf \boxed{\bf x = -2}[/tex]

Convert the following expressions to simplify fraction or integer. If it is not a real number, enter none

Answers

We are given the expression:

[tex]8^{\frac{2}{3}}[/tex]

To get the answer, we will have to apply exponents rules

The rule is:

[tex]a^{\frac{b}{c}}=\sqrt[c]{a}^b[/tex]

Thus

we will have

[tex]8^{\frac{2}{3}}=\sqrt[3]{8^2}=\sqrt[3]{64}=4[/tex]

Therefore,

The answer is 4

evaluate the expression given sin u = 5/13 and cos v = -3/5 where angle u is in quadrant 2and angle v is in quadrant 2sin ( u - v )

Answers

We are given the following information

sin u = 5/13

cos v = -3/5

Where the angle u and v are in the 2nd quadrant.

[tex]\begin{gathered} \cos\theta=\frac{adjacent}{hypotenuse} \\ \sin\theta=\frac{opposite}{hypotenuse} \end{gathered}[/tex]

Let us find cos u

Apply the Pythagorean theorem to find the 3rd side.

[tex]\begin{gathered} a^2+b^2=c^2 \\ a^2=c^2-b^2 \\ a^2=13^2-5^2 \\ a^2=169-25 \\ a^2=144 \\ a=\sqrt{144} \\ a=12 \end{gathered}[/tex]

Cos u = 12/13

Now, let us find sin v

Apply the Pythagorean theorem to find the 3rd side.

[tex]\begin{gathered} a^2+b^2=c^2 \\ b^2=c^2-a^2 \\ b^2=5^2-(-3)^2 \\ b^2=25-9 \\ b^2=16 \\ b=\sqrt{16} \\ b=4 \end{gathered}[/tex]

Sin v = 4/5

Recall the formula for sin (A - B)

[tex]\sin(A-B)=\sin A\cos B-\cos A\sin B[/tex]

Let us apply the above formula to the given expression

[tex]\begin{gathered} \sin(u-v)=\sin u\cdot\cos v+\cos u\cdot\sin v \\ \sin(u-v)=\frac{5}{13}\cdot-\frac{3}{5}+\frac{12}{13}\cdot\frac{4}{5} \\ \sin(u-v)=\frac{33}{65} \end{gathered}[/tex]

Therefore, sin (u - v) = 33/65

For the piecewise function, find the values g(-2), g(2), and g(8).g(x)=X+7, for xs28- x, for x>2.9(-2)=0

Answers

g(x) = x + 7 when x =< 2

g(x) = 8 - x when x > 2

g(-2) Evaluate the first function

g(-2) = -2 + 7

g(-2) = 5

g(2) Evaluate the first function

g(2) = 2 + 7

g(2) = 9

g(8) Evaluate the second function

g(8) = 8 - 8

g(8) = 0

the store bought a bike from the factory for$ 99 and sold I to Andre for $117 what percentage was the markup?

Answers

EXPLANATION

Let's see the facts:

Bike Price: $99

Sold Price: $117

The percentage is given by the following relationship:

[tex]\text{Percentage: }\frac{\text{Selling price per unit}-Cost\text{ price per unit}}{Cost\text{ price per unit}}\cdot100[/tex]

Replacing terms:

[tex]\text{Percentage =}\frac{117-99}{99}\cdot100[/tex][tex]\text{Percentage = 18.18\%}[/tex]

Answer: The markup was 18.18%

In the rectangle below, FH = 4x – 2, EG= 5x-12, and m ZIGF = 53º.Find El and m ZIFE.EFBEI =Хm LIFE =HG

Answers

Answer:

The length EI is;

[tex]EI=19[/tex]

The measure of angle IFE is;

[tex]m\angle IFE=37^{\circ}[/tex]

Explanation:

Given the rectangle in the attached image.

Given;

[tex]\begin{gathered} FH=4x-2 \\ EG=5x-12 \\ m\angle IGF=53^{\circ} \end{gathered}[/tex]

Recall that the length of the diagonals of a rectangle are equal so;

[tex]\begin{gathered} FH=EG \\ 4x-2=5x-12 \end{gathered}[/tex]

solving for x, we have;

[tex]\begin{gathered} 4x-2=5x-12 \\ 12-2=5x-4x \\ x=10 \end{gathered}[/tex]

Since we have the value of x, let us substitute to get the length of diagonal EG;

[tex]\begin{gathered} EG=5x-12 \\ EG=5(10)-12=50-12 \\ EG=38\text{ units} \end{gathered}[/tex]

Also, note that the diagonals of a rectangle bisect each other, so the length of EI would be;

[tex]\begin{gathered} EI=\frac{EG}{2}=\frac{38}{2} \\ EI=19 \end{gathered}[/tex]

Therefore, the length EI is;

[tex]EI=19[/tex]

To get the measure of angle IFE;

[tex]m\angle IGF=m\angle IFG=53^{\circ}[/tex]

Reason: base angles of an isosceles triangle are equal.

So;

[tex]m\angle IFE+m\angle IFG=90^{\circ}[/tex]

Reason: Complementary angles.

Substituting the value of angle IFG;

[tex]\begin{gathered} m\angle IFE+53^{\circ}=90^{\circ} \\ m\angle IFE=90^{\circ}-53^{\circ} \\ m\angle IFE=37^{\circ} \end{gathered}[/tex]

Therefore, the measure of angle IFE is;

[tex]m\angle IFE=37^{\circ}[/tex]

3a + 9 > 21 or -2a +4 > 16

Answers

We will look at how to evaluate inequalities in terms of a number line solution.

We have the following two inequalities:

[tex]3a\text{ + 9 > 21 or -2a + 4 > 16}[/tex]

We will first solve each inequality separately for the variable ( a ) as follows:

[tex]\begin{gathered} 3a\text{ > 12 or -2a > 12} \\ \textcolor{#FF7968}{a}\text{\textcolor{#FF7968}{ > 4 or a < -6}} \end{gathered}[/tex]

Now we will plot the solution on a number line as follows:

10 of 1010 of 10 Items
41:59














Skip to resources
Question
Each equation represents a proportional relationship. Choose the equations for which the constant of proportionality is 14.
Responses
A y = 0.25xy = 0.25x
B 4y = x4y = x
C y = 4xy = 4x
D 32y = 8x32y = 8x
E 14y = 2x

Answers

Answer:

its B

Step-by-step explanation: just took the test

Is there enough information to prove the quadrilateral is a parallelogram if so what property proves it

Answers

To solve this problem we remember the following statement: a quadrilateral that has opposite sides that are congruent and parallel can be a parallelogram, rhombus, rectangle or square.

From the figure, we see a quadrilateral with congruent opposite sides. Relating this information to the statement above, we see that this quadrilateral can be a parallelogram, rhombus, rectangle or square. So we conclude that there is not enough information to conclude that the quadrilateral is a parallelogram.

Answer

d. Not enough information

Which point is located at (-1, -3)?A.point AB.point DC.point ED.point F

Answers

SOLUTION

We are asked which point is located at (-1, -3)

From the diagram given, the point located on (-1, -3) is point E.

Hence option C is the correct answer

i need help with math

Answers

A. ∠4 is congruent to ∠5; True.

B. Two lines are parallel; True.

C. The measure of ∠6 = 90.5°; False.

D. ∠2 and ∠3; True.

What are the properties of angles of parallel lines?On a common plane, two parallel lines do not intersect.As a result, the characteristics of parallel lines with respect to transversals are given below.Angles that correspond are equal.Vertical angles are equal to vertically opposite angles.Interior angles that alternate are equal.The exterior angles that alternate are equal.

For the give question;

Two line are cut by the transversal.

∠1 =  90.5° and ∠7  =  89.5°

Thus the result for the given statement are-

A. ∠4 is congruent to ∠5 because they are alternate interior angles; True.

B. Two lines are parallel; True.

C. The measure of ∠6 = 90.5°; False.

∠6 = ∠7 =  89.5°.(correct)

D. ∠2 and ∠3 are supplementary because they are same-side exterior Angeles; True.

Thus, the result for the given statement are found.

To know more about the parallel lines, here

https://brainly.com/question/4954460

#SPJ1

What is the simplest form of the radical expression? 3 3 √ 2 a − 6 3 √ 2 a
Please show the steps to help me understand this process.

Answers

Simplest form of the  radical expression 3 ∛2 a − 6 ∛2 a is given by -3∛2 a.

As given in the question,

Given radical expression is equal to :

3 ∛2 a − 6 ∛2 a

Simplify the given 3 ∛2 a − 6 ∛2 a radical expression to get the simplest form ,

3 ∛2 a − 6 ∛2 a

Write all the prime factors of the number we have,

= 3∛2 a - ( 3 × 2) ∛2 a

Take out the common factor from the given radical expression we have,

= 3∛2 a ( 1 - 2 )

= 3∛2 a (- 1)

= -3∛2 a

Therefore, simplest form of the  radical expression 3 ∛2 a − 6 ∛2 a is given by -3∛2 a.

The complete question is:

What is the simplest form of the radical expression? 3 ∛2 a − 6 ∛2 a

Please show the steps to help me understand this process.

Learn more about radical expression here

brainly.com/question/3796764

#SPJ1

Find the difference: 8.02 - 0.003A) 7.990B) 8.017C) 8.019D) None of these choices are correct.

Answers

Given:

[tex]8.02-0.003[/tex]

[tex]8.02-0.003=8.017[/tex]

Option B is the final answer.

A triangle has sides measuring 5 inches and 8 inches. If x represents the length in inches of the third side, which inequality gives the range of possible values for x? OA. 3< x< 13 B. 5< x< 8 OC. 3

Answers

Explanation

Step 1

then

If a,b,c are the sides of a triangle it MUST exist these equality:

each side must be less then the sum of the others sides.

[tex]\begin{gathered} then \\ x<5+8 \\ x<13 \\ \end{gathered}[/tex]

Also

[tex]\begin{gathered} x+5>8 \\ x>8-5 \\ x>3 \end{gathered}[/tex]

then,the answer is

[tex]3

Help me with/ #2 plsUsing the graphs what are the solutions to the following systems

Answers

Explanation:

The graph shows a ine crossing the parabola. The solution of the systems is the point where both system of equations intersect.

The line crosses the parabola at two point:

At x = 2, y = 2

This point is applicable to both. Since both have same values at this point, (2, 2) is one of the solution

At x = -2, y = -6

Both graphs have this point . This shows point (-2, -6) is also a solution

Hence, the solutions of the systems are (2, 2) and (-2, -6)

Given the polynomial P(x)= x^3 + 10x^2 + 25xa. List all of the potential rational roots b. Find and list all the actual roots of P(x), and the multiplicity of each root

Answers

a)

In order to find the list of all potential rational roots, let's find the factors of the division between the constant term and the leading term.

Since the constant term is zero, so the only potential rational root in the list is 0.

b)

Since the constant term is zero, so 0 is a root of the polynomial. Then, let's factor it to find the remaining roots:

[tex]\begin{gathered} x^3+10x^2+25x=0 \\ x(x^2+10x+25)=0 \\ x^2+10x+25=0 \end{gathered}[/tex]

Solving this quadratic equation using the quadratic formula, we have:

[tex]\begin{gathered} ax^2+bx+c=0 \\ x^2+10x+25=0 \\ a=1,b=10,c=25 \\ \\ x=\frac{-b\pm\sqrt[]{b^2-4ac}}{2a} \\ x_1=\frac{-10+\sqrt[]{100-100}}{2}=\frac{-10+0}{2}=-5 \\ x_2=\frac{-10-0}{2}=-5 \end{gathered}[/tex]

Therefore the actual roots of P(x) are:

0 (multiplicity 1) and -5 (multiplicity 2).

Hello! I need help solving and answering this practice problem. Having trouble with it.

Answers

In this problem, we have an arithmetic sequence with:

• first term a_1 = -22,

,

• common difference r = 5.

The terms of the arithmetic sequence are given by the following relation:

[tex]a_n=a_1+r\cdot(n-1)\text{.}[/tex]

Replacing the values a_1 = -22 and r = 5, we have:

[tex]a_n=-22+5\cdot(n-1)=-22+5n-5=5n-27.[/tex]

We must compute the sum of the first 30 terms of the sequence.

The sum of the first N terms of a sequence is:

[tex]\begin{gathered} S=\sum ^N_{n\mathop=1}a_n=\sum ^N_{n\mathop{=}1}(5n-27) \\ =5\cdot\sum ^N_{n\mathop{=}1}n-27\cdot\sum ^N_{n\mathop{=}1}1 \\ =5\cdot\frac{N\cdot(N+1)}{2}-27\cdot N. \end{gathered}[/tex]

Where we have used the relations:

[tex]\begin{gathered} \sum ^N_{n\mathop{=}1}n=\frac{N\cdot(N+1)}{2}, \\ \sum ^N_{n\mathop{=}1}1=N\text{.} \end{gathered}[/tex]

Replacing the value N = 30 in the formula for the sum S, we get:

[tex]S=5\cdot\frac{30\cdot31}{2}-27\cdot30=1515.[/tex]

Answer

sum = 1515

Hans cell phone plan cost $200 to start, then there is a $50 charge each month.a. what is the total cost ( start-up fee and monthly charge) to use the cell phone plan for one month? b. what is the total cost for x months?c. graph the cost of the cell phone plan over a. Of 2 years using months as a unit of time. Be sure to scale your access by labeling the grid line with some numbers.(pt2 to letter c) what are the labels for the axes of the graph (for x and y)d. is there a proportional relationship between time and the cost of the cell phone plan?Explain how you know!e. Tyler cell phone plan cost $350 to start then there is a $50 charge each month on the same grid as Hans plan in part C above graph the cost of Tyler cell phone plan over Of 2 yearslastly, describe how hans and Tyler's graphs are similar and how they are different..

Answers

We know that

• The cost is $200 to start and $50 per month. This can be expressed as follows.

[tex]C=200+50m[/tex]

(a) The cost for one month would be

[tex]C=200+50\cdot1=200+50=250[/tex]

(b) The cost for x months is

[tex]C=200+50x[/tex]

(c) To graph the equation, we use the month as a unit of time, the table values would be

m C

1 250

2 300

3 350

4 400

5 450

6 500

7 550

8 600

9 650

10 700

11 750

12 800

Now, we graph all of these points.

The x-axis label is Months, and the y-axis label is Cost.

(d) The given situation does not show a proportional relationship because a proportional relationship is modeled by the form y = kx, which we do not have in this case.

(e) If the initial fee is $350, the equation is

[tex]C=350+50m[/tex]

Let's graph it.

The graphs are similar because they have the same slope but they are different because they have different y-intercepts.

Use the Distributive Property to simplify the following expression.8(x+4)

Answers

Given the expression:

8(x + 4)

Let's simplify using distributive property.

Use distributive property to distribute 8 into x + 4:

8(x) + 8(4)

Evaluate:

8x + 32

ANSWER:

8x + 32

From the entrance, most people will go straight to the roller coaster or straight to the tower. The distance from the entrance to the roller coaster is 461m, and the distance from the entrance to the tower is 707 m. If the paths to these two attractions are separated by a 41o angle, how far apart are the roller coaster and the tower?

Answers

The given situation can be illustrated as follow:

In order to determine the distance x between the roller coaster and the tower. Use the law of cosines, as follow:

[tex]x^2=(461)^2+(707)^2-2(461)(707)\cos 41[/tex]

By simplifying the previous expression, you obtain:

[tex]\begin{gathered} x^2=220409.5413 \\ x=\sqrt[]{220409.5413} \\ x\approx469.5 \end{gathered}[/tex]

Hence, the distance between the tower and the roller coaester is approximately 469.5m

Shelly is rolling a six-sided number cube and recording her results in a chart.Number ofRollsNumber ofTimesLanded on 1Number ofTimesLanded on 2Number ofTimesLanded on 3Number ofTimesLanded on 4Number ofTimesLanded on sNumber ofTimesLanded on 6100141714192019200304237332731300SO54495252600971031051119599AWhich is BEST supported by the data in the chart?А when viewing the data for rolling a one, as the number of rolls Increases, the experimental probability becomes closer to equal to the theoretical probability.when viewing the data for rolling a two, as the number of rolls increases, the experimental probability becomes closer to equal to the theoretical probability.When viewing the data for rolling a four, as the number of rolls increases, the experimental probability becomes closer to equal to the theoretical probability.When viewing the data for rolling a sbc, as the number of rolls increases, the experimental probability becomes closer to equal to the theoretical probability.BСD

Answers

We will have the following:

The expression that best describes the information is:

*When viewing the data for tolling a one, as the number of rolls increases, the experimental probability becomes closer to equal to the theoretical probability.

Emma has money into savings accounts. One rate is 8% and the other is 12%. If she has $450 more in the 12% account and the total interest is $220, how much is invested in each savings account?

Answers

A account 8% B account 12%

A+$450 = B (1)

Ax8% +Bx12%= $220

Ax0.08 + Bx0.12 = 220

Now we replace (1) on B:

Ax0.08 + (A+450)x0.12 = 220

Ax0.08 + Ax0.12 + 54 = 220

Ax0.2 = 166

A= 830.

Now we replace the value of A on equation (1):

830 + 450 = B

B = 1280

NO LINKS I DONT WANT TO DOWNLOAD.

Frankie and Gus swam for 10 minutes. When the time was up, Frankie had completed 10 7/10 laps and Gus 10 4/5 had completed laps. Frankie wrote the inequality 10 7/10 > 10 4/5 to show who swam the longest distance. Was he correct? Explain your answer by describing where the numbers would be positioned on a number line.

Answers

Answer:

Frankie is incorrect.

Step-by-step explanation:

Change 4/5 to 8/10.

You are comparing 10 7/10 to 10 8/10.

10 4/5 (which is also 10 8/10) can be written as 10.8

10 7/10 can be written as 10.7

10.8 > 10.7, so 10 4/5 > 10 7/10 is correct,

and 10 7/10 > 10 4/5 is incorrect.

Frankie is incorrect.

On a number line, show 10 and 11.

Make 10 equal spaces between 10 and 11. Each space is 1/10.

10 7/10 is one space to the left of 10 4/5, so 10 7/10 is less than 10 4/5.

The selling price of a refrigerator is $548.90. If the markup is 10% of the dealer's cost, what is the dealer's cost of the refrigerator?

Answers

Answer

Dealer's cost = $499

Explanation

The markup percent is given as

[tex]\text{Markup percent = }\frac{(Selling\text{ Price) - (Cost)}}{Cost}\times100\text{ percent}[/tex]

Markup percent = 10%

Selling Price = 548.90 dollars

Cost = ?

[tex]\begin{gathered} \text{Markup percent = }\frac{(Selling\text{ Price) - (Cost)}}{Cost}\times100\text{ percent} \\ \text{10 = }\frac{548.90\text{-(Cost)}}{Cost}\times100\text{ percent} \\ 0.1=\frac{548.90-\text{Cost}}{\text{Cost}} \\ \text{Cross multiply} \\ 0.1(\text{Cost) }=548.90-\text{Cost} \end{gathered}[/tex]

0.1 (Cost) + Cost = 548.90

1.1 (Cost) = 548.90

Divide both sides by 1.1

Cost = (548.90/1.1)

Cost = 499 dollars

Hope this Helps!!!

Other Questions
The number of protozoa in a biology laboratory experiment is given by the polynomial functionp(t) = 0.02t^4 + 0.3t^3 + 7t^2, where p is the number of protozoa after t hours.Step 2 of 2: How many protozoa are present after 2 days? Round your answer to the nearest wholenumber. > Next question Get a similar question You can retry this a 8.1 ft 9.7 10.6 Name the Shape: triangle trapezoid kite parallelogram 13. If a trapezoid has base lengths of 18and 40, what is the length of themedian?I Help Dalton explain his work. Complete the paragraph.One way to show that FGHI is a rectangle is to show that all 4 of its angles are rightangles. So, I found the slope of each side. The slope of FG and IH wasTheslope of FI and GH wasThen I knew that ZF, LG, ZH, and ZI were all rightangles becauselines have Malena is in a book club. The graph shows a proportional relationship between the number of the books Malena has read and the number of the years she has been in the club. A. 12in ^2B. 30 in ^2C. 24 in ^2 D. 27 in 2pls help with guided practice How many radians are in a full rotation around a circle, or 360? graph the equation shown below by transforming the given graph of the parent function. y = 2x Perform the indicated operation 4.8L +12.6L 1st. Drop down menu A.DNAB.RNAC.amino acids 2nd drop down menu A.mitochondria B. Nucleus C. RibosomeD. Cytoplasm 3rd drop down menu A. TranslationB. Transcription4th drop down menu A. NucleusB. Cytoplasm5th drop down menu A. TranslationB. Transcription6th drop down menu A. NucleusB. Cytoplasm7th drop down menu A.nucleotides B.amino acids C.proteins 8th drop down menu A.DNA replication B.photosynthesis C. Protein synthesis Re-write 2/5 1/15 as a unit rate Probability of dependent events2/5EspanolA department store is holding a drawing to give away free shopping sprees. There are 9 customers who have entered the drawing: 5 live in the town of Gaston,2 live in Pike, and 2 live in Wells. Two winners will be selected at random. What is the probability that both winners live in Gaston? Write your answer as afraction in simplest form.0DO5? job satisfaction is not how hard one works or how well one works, but rather how much one likes a specific kind of job or work activity. question 1 options: true false Simplify the following union and/or intersection.Answer(-, 3] n [3, 13) The following sequence has a degree of 3:3, -4, -23, -60, -121, -212, -339, ....TrueFalse What is the range of the function?2.-54-3-22-1-2 -3 -4 Jack needs to order some new supplies for the restaurant where he works. The restaurant needs at least 711 glasses. There are currently 206 glasses. If each set on sale contains 10 glasses, write and solve an inequality which can be used to determine ss, the number of sets of glasses Jack could buy for the restaurant to have enough glasses. The Senate in a certain state is comprised of 59 Republicans, 39 Democrats, and 2 Independents. How many committees can be formed if each committee musthave 3 Republicans and 2 Democrats? What is the difference between a repressible operon and an inducible operon? Give an example of each type. You have a coin sitting ona card on top of a glass. You want to put thecoin into the glass, but you are not allowedto pick up the card. Think of how you can dothat. Then write a short explanation of whyit works that would make sense to someonewho doesn't remember Newton's laws ofmotion.